Đến nội dung

Hình ảnh

BẤT ĐẲNG THỨC HƯỚNG TỚI KÌ THI CHUYÊN TOÁN 2021-2022


  • Chủ đề bị khóa Chủ đề bị khóa
Chủ đề này có 90 trả lời

#41
Do Linh An

Do Linh An

    Binh nhì

  • Thành viên mới
  • 14 Bài viết

Topic đã tròn 30 bài, từ bây giờ để các bạn có thể tự luyện tập, sáng tạo với bất đẳng thức, mỗi tuần mình sẽ cố gắng đăng 1 SET BẤT ĐẲNG THỨC gồm 5 bài, các bạn nếu muốn có thể trả lời trong topic, bài nào khó có nhiều bạn hỏi qua tin nhắn cá nhân thì mình sẽ đăng đáp án sau 2 tuần

SET BẤT ĐẲNG THỨC TUẦN 1:

Bài 31: Cho $a,b,c$ là các số thực dương thỏa mãn $abc=1$. Chứng minh rằng: $\sqrt{\frac{ab}{bc^2+1}}+\sqrt{\frac{bc}{ca^2+1}}+\sqrt{\frac{ca}{ab^2+1}}\leqslant \frac{a+b+c}{\sqrt{2}}$

Bài 32: Cho $x,y,z$ là các số thực không âm thỏa mãn $max\left \{ x,y,z \right \}\geqslant 1$. Chứng minh rằng: $x^3+y^3+z^3+(x+y+z-1)^2\geqslant 1+3xyz$

Chú ý: $max\left \{ x,y,z \right \}$ là số lớn nhất trong 3 số $x,y,z$

Bài 33: Cho $a,b,c$ là các số thực không âm thỏa mãn $(a+b)(b+c)(c+a)>0$. Chứng minh rằng: $\sqrt{\frac{a}{b+c}}+\sqrt{\frac{b}{c+a}}+\sqrt{\frac{c}{a+b}}+\frac{4\sqrt{ab+bc+ca}}{a+b+c}\geqslant 4$

Bài 34: Cho $a,b,c$ là các số thực dương thỏa mãn $a+b+c=ab+bc+ca$. Chứng minh rằng: $\frac{a^2}{b^3+b^2c}+\frac{b^2}{c^3+c^2a}+\frac{c^2}{a^3+a^2b}\geqslant \frac{a+b+c}{2}$

                                                                       (96+ ĐỀ ÔN LUYỆN CHUYÊN TOÁN - VÕ QUỐC BÁ CẨN)

Bài 35: Cho $a,b,c$ là các số thực dương. Chứng minh rằng: $ab\sqrt{ab}+bc\sqrt{bc}+ca\sqrt{ca}\leqslant abc+\frac{1}{2}\sqrt[3]{\frac{(a^2+bc)^2(b^2+ca)^2(c^2+ab)^2}{abc}}$

Bài 34:

Theo bất đẳng thức Bunhiacopxki:

$[\dfrac{a^2}{b^2(b+c)}+\dfrac{b^2}{c^2(c+a)}+\dfrac{c^2}{a^2(a+b)}][(b+c)+(c+a)+(a+b)]\ge (\dfrac{a}{b}+\dfrac{b}{c}+\dfrac{c}{a})^2 \ge (\dfrac{(a+b+c)^2}{a+b+c})^2=(a+b+c)^2$

$\Rightarrow VT\ge \dfrac{(a+b+c)^2}{b+c+c+a+a+b}=\dfrac{a+b+c}{2}$

Dấu bằng xảy ra khi $a=b=c=1$


Bài viết đã được chỉnh sửa nội dung bởi KietLW9: 17-03-2022 - 18:40


#42
KietLW9

KietLW9

    Đại úy

  • Điều hành viên THCS
  • 1737 Bài viết

Bài 34:

Theo bất đẳng thức Bunhiacopxki:

$[\dfrac{a^2}{b^2(b+c)}+\dfrac{b^2}{c^2(c+a)}+\dfrac{c^2}{a^2(a+b)}][(b+c)+(c+a)+(a+b)]\ge (\dfrac{a}{b}+\dfrac{b}{c}+\dfrac{c}{a})^2 \ge (\dfrac{(a+b+c)^2}{a+b+c})^2=(a+b+c)^2$

$\Rightarrow VT\ge \dfrac{(a+b+c)^2}{b+c+c+a+a+b}=\dfrac{a+b+c}{2}$

Dấu bằng xảy ra khi $a=b=c=1$

Hmm có lẽ đây là bài dễ nhất trong SET này, cách của mình:

Lời giải.

Áp dụng bất đẳng thức Cauchy-Schwarz: $\frac{a^2}{b^3+b^2c}+\frac{b^2}{c^3+c^2a}+\frac{c^2}{a^3+a^2b}=\frac{a^4c^2}{a^2c^2b^3+a^2b^2c^3}+\frac{b^4a^2}{c^3b^2a^2+b^2c^2a^3}+\frac{c^4b^2}{c^2b^2a^3+c^2a^2b^3}\geqslant \frac{(a^2c+b^2a+c^2b)^2}{2a^2b^2c^2(a+b+c)}$

Ta cần chứng minh: $ab^2+bc^2+ca^2\geqslant abc(a+b+c)\Leftrightarrow \frac{a}{b}+\frac{b}{c}+\frac{c}{a}\geqslant a+b+c$

Bất đẳng thức cuối đúng do: $\frac{a}{b}+\frac{b}{c}+\frac{c}{a}\geqslant \frac{(a+b+c)^2}{ab+bc+ca}=a+b+c$

Đẳng thức xảy ra khi $a=b=c=1$


Trong cuộc sống không có gì là đẳng thức , tất cả đều là bất đẳng thức  :ukliam2:   :ukliam2: 

 

 

$\text{LOVE}(\text{KT}) S_a (b - c)^2 + S_b (c - a)^2 + S_c (a - b)^2 \geqslant 0\forall S_a,S_b,S_c\geqslant 0$

 

 

 


#43
Do Linh An

Do Linh An

    Binh nhì

  • Thành viên mới
  • 14 Bài viết

Topic đã tròn 30 bài, từ bây giờ để các bạn có thể tự luyện tập, sáng tạo với bất đẳng thức, mỗi tuần mình sẽ cố gắng đăng 1 SET BẤT ĐẲNG THỨC gồm 5 bài, các bạn nếu muốn có thể trả lời trong topic, bài nào khó có nhiều bạn hỏi qua tin nhắn cá nhân thì mình sẽ đăng đáp án sau 2 tuần

SET BẤT ĐẲNG THỨC TUẦN 1:

Bài 31: Cho $a,b,c$ là các số thực dương thỏa mãn $abc=1$. Chứng minh rằng: $\sqrt{\frac{ab}{bc^2+1}}+\sqrt{\frac{bc}{ca^2+1}}+\sqrt{\frac{ca}{ab^2+1}}\leqslant \frac{a+b+c}{\sqrt{2}}$

Bài 32: Cho $x,y,z$ là các số thực không âm thỏa mãn $max\left \{ x,y,z \right \}\geqslant 1$. Chứng minh rằng: $x^3+y^3+z^3+(x+y+z-1)^2\geqslant 1+3xyz$

Chú ý: $max\left \{ x,y,z \right \}$ là số lớn nhất trong 3 số $x,y,z$

Bài 33: Cho $a,b,c$ là các số thực không âm thỏa mãn $(a+b)(b+c)(c+a)>0$. Chứng minh rằng: $\sqrt{\frac{a}{b+c}}+\sqrt{\frac{b}{c+a}}+\sqrt{\frac{c}{a+b}}+\frac{4\sqrt{ab+bc+ca}}{a+b+c}\geqslant 4$

Bài 34: Cho $a,b,c$ là các số thực dương thỏa mãn $a+b+c=ab+bc+ca$. Chứng minh rằng: $\frac{a^2}{b^3+b^2c}+\frac{b^2}{c^3+c^2a}+\frac{c^2}{a^3+a^2b}\geqslant \frac{a+b+c}{2}$

                                                                       (96+ ĐỀ ÔN LUYỆN CHUYÊN TOÁN - VÕ QUỐC BÁ CẨN)

Bài 35: Cho $a,b,c$ là các số thực dương. Chứng minh rằng: $ab\sqrt{ab}+bc\sqrt{bc}+ca\sqrt{ca}\leqslant abc+\frac{1}{2}\sqrt[3]{\frac{(a^2+bc)^2(b^2+ca)^2(c^2+ab)^2}{abc}}$

Bài 32:

Giả sử $x$ là số lớn nhất tức $x\ge1$

$x^3+y^3+z^3+(x+y+z-1)^2-1-3xyz\\=x^3+(y+z)^3-3yz(y+z+x)+(x+y+z)^2-2(x+y+z)\\=(x+y+z)[x^2-x(y+z)+(y+z)^2-2+x+y+z]-3yz(x+y+z)\\\ge (x+y+z)[x^2-x(y+z)+(y+z)^2-2+x+y+z]-\dfrac{3}{4}(y+z)^2(x+y+z)\\=(x+y+z)[x^2-x(y+z)-2+x+y+z+(y+z)^2-\dfrac{3}{4}(y+z)^2]\\=\dfrac{1}{4}(x+y+z)(4x^2-4x(y+z)-8+4x+4y+4z+(y+z)^2]\\=\dfrac{1}{4}(x+y+z)[(2x-y-z)^2 +4x+4y+4z-8]\\=\dfrac{1}{4}(x+y+z)[(y+z-2x)^2+4(y+z-2x)+4+12x-12]\\=\dfrac{1}{4}(x+y+z)[(y+z-2x+2)^2+12(x-1)]\ge0\\\Rightarrow x^3+y^3+z^3+(x+y+z-1)^2\ge1+3xyz$

Dấu bằng xảy ra khi $(x;y;z)=(1;0;0)$ và các hoán vị


Bài viết đã được chỉnh sửa nội dung bởi KietLW9: 19-03-2022 - 12:04


#44
KietLW9

KietLW9

    Đại úy

  • Điều hành viên THCS
  • 1737 Bài viết

Bài 32:

Giả sử $x$ là số lớn nhất tức $x\ge1$

$x^3+y^3+z^3+(x+y+z-1)^2-1-3xyz\\=x^3+(y+z)^3-3yz(y+z+x)+(x+y+z)^2-2(x+y+z)\\=(x+y+z)[x^2-x(y+z)+(y+z)^2-2+x+y+z]-3yz(x+y+z)\\\ge (x+y+z)[x^2-x(y+z)+(y+z)^2-2+x+y+z]-\dfrac{3}{4}(y+z)^2(x+y+z)\\=(x+y+z)[x^2-x(y+z)-2+x+y+z+(y+z)^2-\dfrac{3}{4}(y+z)^2]\\=\dfrac{1}{4}(x+y+z)(4x^2-4x(y+z)-8+4x+4y+4z+(y+z)^2]\\=\dfrac{1}{4}(x+y+z)[(2x-y-z)^2 +4x+4y+4z-8]\\=\dfrac{1}{4}(x+y+z)[(y+z-2x)^2+4(y+z-2x)+4+12x-12]\\=\dfrac{1}{4}(x+y+z)[(y+z-2x+2)^2+12(x-1)]\ge0\\\Rightarrow x^3+y^3+z^3+(x+y+z-1)^2\ge1+3xyz$

Dấu bằng xảy ra khi $(x;y;z)=(1;0;0)$ và các hoán vị

Bài này có một biến đổi đơn giản hơn một chút như sau :D 

Lời giải.

Từ giả thiết ta có $x+y+z>0$ nên bất đẳng thức cần chứng minh tương đương: $x^2+y^2+z^2-xy-yz-zx+x+y+z-2\geqslant 0$

Giả sử $x$ là số lớn nhất trong 3 số $x,y,z$ thì $x\geqslant 1$

Như vậy ta cần chứng minh: $(x-y-1)^2+(x-z-1)^2+(y-z)^2+6(x-1)\geqslant 0$

Đẳng thức xảy ra khi $x=1,y=z=0$ và các hoán vị


Trong cuộc sống không có gì là đẳng thức , tất cả đều là bất đẳng thức  :ukliam2:   :ukliam2: 

 

 

$\text{LOVE}(\text{KT}) S_a (b - c)^2 + S_b (c - a)^2 + S_c (a - b)^2 \geqslant 0\forall S_a,S_b,S_c\geqslant 0$

 

 

 


#45
KietLW9

KietLW9

    Đại úy

  • Điều hành viên THCS
  • 1737 Bài viết

Trước khi đến các bất đẳng thức trong SET tuần 2 thì mình xin chia sẻ tiếp 2 bài bất đẳng thức khá hay!

Bài 36: Cho $a,b,c$ là các số không âm thỏa mãn $x+y+z=2$. Chứng minh rằng: $(3x^2-2xy+3y^2)(3y^2-2yz+3z^2)(3z^2-2zx+3x^2)\leqslant 36$

~~~~~~~~~~~~~~~~~~~~

Lời giải.

Vì bất đẳng thức mang tính đối xứng giữa các biến nên ta giả sử $x\geqslant y\geqslant z$

Lúc này áp dụng bất đẳng thức $AM-GM$, ta được: $(3x^2-2xy+3y^2)(3y^2-2yz+3z^2)(3z^2-2zx+3x^2)\leqslant (3x^2-2xy+3y^2)(3y^2+yz)(3x^2+zx)=xy(3y+z)(3x+z)(3x^2-2xy+3y^2)=\frac{1}{4}.4xy(9xy+3zx+3yz+z^2)(3x^2-2xy+3y^2)=\frac{9}{16}.4xy(4xy+\frac{4}{3}zx+\frac{4}{3}yz+\frac{4}{9}z^2)(3x^2-2xy+3y^2)\leqslant \frac{9}{16}.4xy(4xy+6zx+6yz+3z^2)(3x^2-2xy+3y^2)\leqslant \frac{9}{16}.\frac{(3x^2+3y^2+3z^2+6xy+6yz+6zx)^3}{27}=36$

Đẳng thức xảy ra khi $x=y=1, z=0$ và các hoán vị

Bài 37: Cho $a,b,c$ là các số thực không âm thỏa mãn $a+b+c=\frac{4}{3}$. Chứng minh rằng: $3\left [ a(a-1)^2+b(b-1)^2+c(c-1)^2 \right ]\geqslant ab+bc+ca$

~~~~~~~~~~~~~~~~~~~~

Lời giải.

Biến đổi vế trái: $3\left [ a(a-1)^2+b(b-1)^2+c(c-1)^2 \right ]=3\left [ (a^3+b^3+c^3)-2(a^2+b^2+c^2)+\frac{4}{3} \right ]=3\left [ (a+b+c)(a^2+b^2+c^2-ab-bc-ca)+3abc-2(a^2+b^2+c^2)+\frac{4}{3} \right ]=3\left [ \frac{2}{3}\left [ 3(a^2+b^2+c^2)-(a+b+c)^2 \right ]+3abc-2(a^2+b^2+c^2)+\frac{4}{3} \right ]=3\left [ \frac{4}{27}+3abc \right ]=9abc+\frac{4}{9}$

Vậy ta cần chứng minh: $9abc+\frac{4}{9}\geqslant ab+bc+ca$

Giả sử $a$ là số lớn nhất trong 3 số $a,b,c$ thì bất đẳng thức tương đương: $\frac{4}{9}-a(b+c)+bc(9a-1)\geqslant 0$

Đúng do $9a-1>0$ theo giả sử và $\frac{4}{9}-a(b+c)=\frac{4}{9}-a(\frac{4}{3}-a)=(a-\frac{2}{3})^2\geqslant 0$

Đẳng thức xảy ra khi $a=b=\frac{2}{3},c=0$ và các hoán vị

~~~~~~~~~~~~~~~~~~~~~~~~~~~~~~~~~~~~~~~~~~~~~~~~~~~~~~~~~~~~~~~~~~~~~~~~~~~~~~~~~~~~~~~~~~~~~~~~~~~~~~~~~~~~~~~~~~~~~~~

Mình tiếp tục xin chia sẻ với các bạn các mẹo nhỏ trong bất đẳng thức

1) Khi gặp giả thiết $a+b+c+1=4abc$ thì việc đầu tiên ta nghĩ đến là biến đổi cái này thành dạng: $\frac{1}{2a+1}+\frac{1}{2b+1}+\frac{1}{2c+1}=1$ sau đó sẽ phát triển hướng, nhưng không phải bài nào giả thiết này cũng biến đổi như vậy, vẫn có vài bài không cần!

2) Một đẳng thức rất mạnh và hữu hiệu: $(a^2+b^2+c^2)(ab+bc+ca)^2-(a^2+2bc)(b^2+2ca)(c^2+2ab)=(a-b)^2(b-c)^2(c-a)^2$


Trong cuộc sống không có gì là đẳng thức , tất cả đều là bất đẳng thức  :ukliam2:   :ukliam2: 

 

 

$\text{LOVE}(\text{KT}) S_a (b - c)^2 + S_b (c - a)^2 + S_c (a - b)^2 \geqslant 0\forall S_a,S_b,S_c\geqslant 0$

 

 

 


#46
PDF

PDF

    Trung sĩ

  • Thành viên
  • 197 Bài viết

Một bài vui vẻ mà tôi bịa ra, đóng góp cho topic:

$\textbf{Bài 38.}$ Cho $a,b,c$ thực, không đồng thời bằng $0$.

Tìm giá trị nhỏ nhất và giá trị lớn nhất của

$$P=\frac{bc}{a^{2}+2b^{2}+2c^{2}}+\frac{ca}{b^{2}+2c^{2}+2a^{2}}+\frac{ab}{c^{2}+2a^{2}+2b^{2}}.$$



#47
KietLW9

KietLW9

    Đại úy

  • Điều hành viên THCS
  • 1737 Bài viết

Tổng hợp ba bài bất đẳng thức đặc sắc trong TẠP CHÍ TOÁN TUỔI THƠ năm 2021

Bài 38: Cho $a,b,c$ là các số thực thỏa mãn $abc=-2$ và $a+b+c=0$. Chứng minh rằng: $\frac{ab+bc+ca-a^2-b^2-c^2}{a^3+b^3+c^3}\geqslant  \frac{3}{2}$

~~~~~~~~~~~~~~~~~~~

Lời giải.

Áp dụng bất đẳng thức $AM-GM$, ta được: $\frac{ab+bc+ca-a^2-b^2-c^2}{a^3+b^3+c^3}=\frac{3a(b+c)+3bc-(a+b+c)^2}{a^3+(b+c)^3-3bc(b+c)}=\frac{-3a^2+3bc}{3abc}=\frac{a^2+\frac{2}{a}}{2}=\frac{1}{2}\left [ (a-1)^2+\frac{2(a-1)^2}{a}+3 \right ]\geqslant \frac{3}{2}$

Đẳng thức xảy ra khi $a=b=1,c=-2$ và các hoán vị

Bài 39: Cho $r\geqslant 1$. Chứng minh rằng: $(a^2+r)(b^2+r)(c^2+r)\geqslant (r+1)^2(ab+bc+ca+r-2)$

~~~~~~~~~~~~~~~~~~~

Lời giải.

Ta có: $(a^2+r)(b^2+r)=a^2b^2+r(a^2+b^2)+r^2=(a^2b^2-2ab+1)+(a^2+b^2+2ab)-(a^2+b^2)+(r-1)(a^2+b^2)+r^2-1=(ab-1)^2+(a+b)^2+\frac{r-1}{2}\left [ (a+b)^2+(a-b)^2 \right ]+(r+1)(r-1)\geqslant \frac{r+1}{2}(a+b)^2+(r+1)(r-1)=(r+1)\left [ \frac{(a+b)^2}{2}+r-1 \right ]$

Ta cần chứng minh: $\left [ \frac{(a+b)^2}{2}+r-1 \right ](c^2+r)\geqslant (r+1)(ab+bc+ca+r-2)$

Thật vậy, ta có: $\left [ \frac{(a+b)^2}{2}+r-1 \right ](c^2+r)- (r+1)(ab+bc+ca+r-2)=\frac{(a+b)^2c^2}{2}+c^2(r-1)+\frac{r(a+b)^2}{2}+(r^2-r)-(r+1)ab-c(a+b)(r+1)-(r^2-r-2)=\left [ \frac{(a+b)^2c^2}{2}-2c(a+b)+2  \right ]-(r-1)c(a+b)+c^2(r-1)+\frac{r(a^2+b^2)}{2}+rab-(r+1)ab=\frac{(ac+bc-2)^2}{2}+\frac{r-1}{2}\left [ 2c^2-2ac-2bc \right ]+\frac{r-1}{2}(a^2+b^2)+\frac{a^2+b^2}{2}-ab=\frac{(ac+bc-2)^2}{2}+\frac{r-1}{2}\left [ (a-c)^2+(b-c)^2 \right ]+\frac{(a-b)^2}{2}\geqslant 0$

Vậy bất đẳng thức được chứng minh

Đẳng thức xảy ra khi $a=b=c=1$ hoặc $a=b=c=-1$

Bài 40: Cho $a,b,c$ là các số thực dương thỏa mãn $x^2+2x(y+z)=5yz$. Chứng minh rằng: $(x+y)^3+(x+z)^3+(x+y)(y+z)(z+x)\leqslant 3(y+z)^3$

~~~~~~~~~~~~~~~~

Lời giải.

Từ giả thiết ta có: $x^2+2x(y+z)=5yz\leqslant \frac{5}{4}(y+z)^2\Rightarrow 2x\leqslant y+z$

Ta cần chứng minh: $x^3+2xy(x+y)+2xz(x+z)+xyz\leqslant (y+z)(y^2+3yz+z^2)\Leftrightarrow x\left [ x^2+2xy+2xz+2y^2+2z^2+yz \right ]\leqslant (y+z)(y^2+3yz+z^2)\Leftrightarrow x\left [ 2y^2+2z^2 +6yz\right ]\leqslant (y+z)(y^2+3yz+z^2)\Leftrightarrow 2x\leqslant y+z$

Bất đẳng thức cuối đúng nên ta có điều phải chứng minh.


Bài viết đã được chỉnh sửa nội dung bởi KietLW9: 20-03-2022 - 09:59

Trong cuộc sống không có gì là đẳng thức , tất cả đều là bất đẳng thức  :ukliam2:   :ukliam2: 

 

 

$\text{LOVE}(\text{KT}) S_a (b - c)^2 + S_b (c - a)^2 + S_c (a - b)^2 \geqslant 0\forall S_a,S_b,S_c\geqslant 0$

 

 

 


#48
KietLW9

KietLW9

    Đại úy

  • Điều hành viên THCS
  • 1737 Bài viết

SET BẤT ĐẲNG THỨC TUẦN 2:

Bài 41: Cho $a,b,c$ là các số thực dương thỏa mãn $ab+bc+ca=\frac{1}{ab}+\frac{1}{bc}+\frac{1}{ca}$. Chứng minh rằng: $a(a+1)+b(b+1)+c(c+1)\geqslant \sqrt{\frac{2(a^2bc+1)}{a}}+\sqrt{\frac{2(b^2ca+1)}{b}}+\sqrt{\frac{2(c^2ab+1)}{c}}$

Bài 42: Cho $a,b,c,d$ là các số thực dương thỏa mãn $a+b+c+d=4$. Chứng minh rằng: $\frac{1}{ab}+\frac{1}{cd}\geqslant \frac{a^2+b^2+c^2+d^2}{2}$

Bài 43: Cho $a,b,c$ là các số thực không âm thỏa mãn $a+b+c=3, a^2+b^2+c^2=5$. Chứng minh rằng: $a^3b+b^3c+c^3a\leqslant 8$

Bài 44: Cho $a,b,c$ là các số thực không âm thỏa mãn $ab+bc+ca+abc\leqslant 4$. Chứng minh rằng: $\frac{13a^2+2a}{a^2+a+1}+\frac{13b^2+2b}{b^2+b+1}+\frac{13c^2+2c}{c^2+c+1}\leqslant 16$

Bài 45: Cho $a,b,c$ là các số thực dương thỏa mãn $ \sqrt{a}+ \sqrt{b}+ \sqrt{c}=3$. Chứng minh rằng: $$\frac{1}{a^2+b^2}+\frac{1}{b^2+c^2}+\frac{1}{c^2+a^2}+\frac{5}{2}(a+b+c)\geqslant 9$$


Trong cuộc sống không có gì là đẳng thức , tất cả đều là bất đẳng thức  :ukliam2:   :ukliam2: 

 

 

$\text{LOVE}(\text{KT}) S_a (b - c)^2 + S_b (c - a)^2 + S_c (a - b)^2 \geqslant 0\forall S_a,S_b,S_c\geqslant 0$

 

 

 


#49
Hoang72

Hoang72

    Thiếu úy

  • Điều hành viên OLYMPIC
  • 539 Bài viết

Bài 43: Cho $a,b,c$ là các số thực không âm thỏa mãn $a+b+c=3, a^2+b^2+c^2=5$. Chứng minh rằng: $a^3b+b^3c+c^3a\leqslant 8$

Đặt $M=a^3b+b^3c+c^3a;N=ab^3+bc^3+ca^3$.

Ta có $M+N\leq (ab+bc+ca)(a^2+b^2+c^2)=10$.

Xét hằng đẳng thức $M-N=(a+b+c)(a-b)(b-c)(a-c)$.

Giả sử $a$ là số lớn nhất.

+) Nếu $a\geq b\geq c$ thì ta có $\frac{3}{2}(a-c)^2\leq (a-b)^2+(b-c)^2+(c-a)^2=6\Rightarrow a-c\leq 2$.

Do đó $M-N=3(a-b)(b-c)(a-c)\leq \frac{3}{4}(a-c)^3\leq 6$.

Suy ra $2M=M-N+M+N\leq 16$ nên $M\leq 8$.

+) Nếu $a\geq c\geq b$ thì $M\leq N\Rightarrow M\leq 5<8$.

BĐT được chứng minh. Dấu bằng xảy ra khi $a=2;b=1;c=0$.



#50
KietLW9

KietLW9

    Đại úy

  • Điều hành viên THCS
  • 1737 Bài viết

Đặt $M=a^3b+b^3c+c^3a;N=ab^3+bc^3+ca^3$.

Ta có $M+N\leq (ab+bc+ca)(a^2+b^2+c^2)=10$.

Xét hằng đẳng thức $M-N=(a+b+c)(a-b)(b-c)(a-c)$.

Giả sử $a$ là số lớn nhất.

+) Nếu $a\geq b\geq c$ thì ta có $\frac{3}{2}(a-c)^2\leq (a-b)^2+(b-c)^2+(c-a)^2=6\Rightarrow a-c\leq 2$.

Do đó $M-N=3(a-b)(b-c)(a-c)\leq \frac{3}{4}(a-c)^3\leq 6$.

Suy ra $2M=M-N+M+N\leq 16$ nên $M\leq 8$.

+) Nếu $a\geq c\geq b$ thì $M\leq N\Rightarrow M\leq 5<8$.

BĐT được chứng minh. Dấu bằng xảy ra khi $a=2;b=1;c=0$.

Có một cách dồn về một biến  :icon6:

Lời giải.

Giả sử $a=max\left \{ a,b,c \right \}$ thì $a^2=5-(b+c)^2+2bc\geqslant 5-(3-a)^2\Rightarrow 2a^2-6a+4\geqslant 0\Rightarrow a\geqslant 2\Rightarrow b+c\leqslant 1\Rightarrow b,c\leqslant 1$

Mặt khác từ giả thiết cũng có: $ab+bc+ca=2$

Vậy ta được: $a^3b+b^3c+c^3a\leqslant a^3b+bc+ca=a^3b+2-ab=ab(a^2-1)+2$

Mà $ab=\frac{1}{2}.a.2b\leqslant \frac{a^2+4b^2}{4}\leqslant \frac{5+3b^2}{4}$ và $a^2-1\leqslant 4-b^2$ nên ta cần chứng minh:$\frac{(3b^2+5)(4-b^2)}{4}+2\leqslant 8\Leftrightarrow (b-1)(b+1)(3b^2-4)\geqslant 0$ 

Bất đẳng thức cuối đúng do $b\leqslant 1$

Đẳng thức xảy ra khi $a=2,b=1,c=0$ và các hoán vị


Trong cuộc sống không có gì là đẳng thức , tất cả đều là bất đẳng thức  :ukliam2:   :ukliam2: 

 

 

$\text{LOVE}(\text{KT}) S_a (b - c)^2 + S_b (c - a)^2 + S_c (a - b)^2 \geqslant 0\forall S_a,S_b,S_c\geqslant 0$

 

 

 


#51
Do Linh An

Do Linh An

    Binh nhì

  • Thành viên mới
  • 14 Bài viết

Topic đã tròn 30 bài, từ bây giờ để các bạn có thể tự luyện tập, sáng tạo với bất đẳng thức, mỗi tuần mình sẽ cố gắng đăng 1 SET BẤT ĐẲNG THỨC gồm 5 bài, các bạn nếu muốn có thể trả lời trong topic, bài nào khó có nhiều bạn hỏi qua tin nhắn cá nhân thì mình sẽ đăng đáp án sau 2 tuần

SET BẤT ĐẲNG THỨC TUẦN 1:

Bài 31: Cho $a,b,c$ là các số thực dương thỏa mãn $abc=1$. Chứng minh rằng: $\sqrt{\frac{ab}{bc^2+1}}+\sqrt{\frac{bc}{ca^2+1}}+\sqrt{\frac{ca}{ab^2+1}}\leqslant \frac{a+b+c}{\sqrt{2}}$

Bài 32: Cho $x,y,z$ là các số thực không âm thỏa mãn $max\left \{ x,y,z \right \}\geqslant 1$. Chứng minh rằng: $x^3+y^3+z^3+(x+y+z-1)^2\geqslant 1+3xyz$

Chú ý: $max\left \{ x,y,z \right \}$ là số lớn nhất trong 3 số $x,y,z$

Bài 33: Cho $a,b,c$ là các số thực không âm thỏa mãn $(a+b)(b+c)(c+a)>0$. Chứng minh rằng: $\sqrt{\frac{a}{b+c}}+\sqrt{\frac{b}{c+a}}+\sqrt{\frac{c}{a+b}}+\frac{4\sqrt{ab+bc+ca}}{a+b+c}\geqslant 4$

Bài 34: Cho $a,b,c$ là các số thực dương thỏa mãn $a+b+c=ab+bc+ca$. Chứng minh rằng: $\frac{a^2}{b^3+b^2c}+\frac{b^2}{c^3+c^2a}+\frac{c^2}{a^3+a^2b}\geqslant \frac{a+b+c}{2}$

                                                                       (96+ ĐỀ ÔN LUYỆN CHUYÊN TOÁN - VÕ QUỐC BÁ CẨN)

Bài 35: Cho $a,b,c$ là các số thực dương. Chứng minh rằng: $ab\sqrt{ab}+bc\sqrt{bc}+ca\sqrt{ca}\leqslant abc+\frac{1}{2}\sqrt[3]{\frac{(a^2+bc)^2(b^2+ca)^2(c^2+ab)^2}{abc}}$

Bài 31: 

Đặt $a=\dfrac{1}{x};b=\dfrac{1}{y};c=\dfrac{1}{z} \Rightarrow xyz=1$

$\Rightarrow \sqrt{\dfrac{ab}{bc^2+1}}=\sqrt{\dfrac{ab}{bc(c+a)}}=\sqrt{\dfrac{a}{c(c+a)}}=\dfrac{x}{\sqrt{x+y}}$

Tương tự, có:

$VT=\dfrac{x}{\sqrt{x+y}}+\dfrac{y}{\sqrt{y+z}}+\dfrac{z}{\sqrt{z+x}}$

$\le \sqrt{(x+y+z)(\dfrac{x}{x+y}+\dfrac{y}{y+z}+\dfrac{z}{z+x})}$

$=\sqrt{\dfrac{(x+y+z) x}{x+y}+\dfrac{(x+y+z) y}{y+z}+\dfrac{(x+y+z) z}{z+x}}$

$=\sqrt{x+\dfrac{z x}{x+y}+y+\dfrac{x y}{y+z}+z+\dfrac{y z}{z+x}}$

$=\sqrt{x+\dfrac{z x y}{(x+y) y}+y+\dfrac{x y z}{(y+z) z}+z+\dfrac{y z x}{(z+x) x}}$

$\le \sqrt{x+\dfrac{z  \frac{1}{4}(x+y)^{2}}{(x+y) y}+y+\dfrac{x  \frac{1}{4}(y+z)^{2}}{(y+z) z}+z+\dfrac{y  \frac{1}{4}(z+x)^{2}}{(z+x) x}}$

$=\sqrt{x+\dfrac{z(x+y)}{4 y}+y+\dfrac{x(y+z)}{4 z}+z+\dfrac{y(z+x)}{4 x}}$

$=\sqrt{\dfrac{1}{4}\left(\dfrac{x y}{z}+\dfrac{y z}{x}+\dfrac{z x}{y}\right)+\frac{5}{4}(x+y+z)}$

$=\sqrt{\dfrac{1}{4}\left[(x y)^{2}+(y z)^{2}+(z x)^{2}\right]+\dfrac{5}{4}(x+y+z) x y z}$

 

Đến đây ta cần chứng minh $VP=\dfrac{a+b+c}{\sqrt{2}}=\dfrac{xy+yz+zx}{\sqrt{2}}\ge\sqrt{\dfrac{1}{4}\left[(x y)^{2}+(y z)^{2}+(z x)^{2}\right]+\dfrac{5}{4}(x+y+z) x y z}$

$\Rightarrow \dfrac{(xy+yz+zx)^2}{2} \ge \dfrac{1}{4}\left[(x y)^{2}+(y z)^{2}+(z x)^{2}\right]+\dfrac{5}{4}(x+y+z) x y z$

$\Rightarrow (xy)^2+(yz)^2+(zx)^2 \ge xy.yz+yz.zx+zx.xy$ (luôn đúng)

Dấu bằng xảy ra khi $a=b=c=1$


Bài viết đã được chỉnh sửa nội dung bởi KietLW9: 22-03-2022 - 22:20


#52
Do Linh An

Do Linh An

    Binh nhì

  • Thành viên mới
  • 14 Bài viết

SET BẤT ĐẲNG THỨC TUẦN 2:

Bài 41: Cho $a,b,c$ là các số thực dương thỏa mãn $ab+bc+ca=\frac{1}{ab}+\frac{1}{bc}+\frac{1}{ca}$. Chứng minh rằng: $a(a+1)+b(b+1)+c(c+1)\geqslant \sqrt{\frac{2(a^2bc+1)}{a}}+\sqrt{\frac{2(b^2ca+1)}{b}}+\sqrt{\frac{2(c^2ab+1)}{c}}$

Bài 42: Cho $a,b,c,d$ là các số thực dương thỏa mãn $a+b+c+d=4$. Chứng minh rằng: $\frac{1}{ab}+\frac{1}{cd}\geqslant \frac{a^2+b^2+c^2+d^2}{2}$

Bài 43: Cho $a,b,c$ là các số thực không âm thỏa mãn $a+b+c=3, a^2+b^2+c^2=5$. Chứng minh rằng: $a^3b+b^3c+c^3a\leqslant 8$

Bài 44: Cho $a,b,c$ là các số thực không âm thỏa mãn $ab+bc+ca+abc\leqslant 4$. Chứng minh rằng: $\frac{13a^2+2a}{a^2+a+1}+\frac{13b^2+2b}{b^2+b+1}+\frac{13c^2+2c}{c^2+c+1}\leqslant 16$

Bài 45: Cho $a,b,c$ là các số thực dương thỏa mãn $ \sqrt{a}+ \sqrt{b}+ \sqrt{c}=3$. Chứng minh rằng: $$\frac{1}{a^2+b^2}+\frac{1}{b^2+c^2}+\frac{1}{c^2+a^2}+\frac{5}{2}(a+b+c)\geqslant 9$$

Bài 42: Gặp rồi

Giả sử $a+b\le c+d$

$\dfrac{1}{ab}+\dfrac{1}{cd}\ge\dfrac{a^2+b^2+c^2+d^2}{2}$

$\Rightarrow \dfrac{1}{ab}-2+ab+\dfrac{1}{cd}-2+cd\ge\dfrac{a^2+b^2+c^2+c^2+2ab+2cd}{2}-4$

$\Rightarrow (\dfrac{1}{\sqrt{ab}}-\sqrt{ab})^2+(\dfrac{1}{\sqrt{cd}}-\sqrt{cd})^2\ge \dfrac{(a+b)^2+(c+d)^2}{2}-\dfrac{(a+b+c+d)^2}{4}$

$\Rightarrow (\dfrac{1}{\sqrt{ab}}-\sqrt{ab})^2 \ge \dfrac{(a+b-c-d)^2}{4}$

Vì $a+b\le c+d \Rightarrow a+b+c+d\le0;2\ge a+b \ge 2\sqrt{ab} \Rightarrow \sqrt{ab}\le 1 \Rightarrow \dfrac{1}{\sqrt{ab}}-\sqrt{ab} \ge0 $

Vậy ta cần chứng minh:

$\dfrac{1}{\sqrt{ab}}-\sqrt{ab} \ge \dfrac{c+d-a-b}{2}$

$\Rightarrow \dfrac{1}{\sqrt{ab}}-\sqrt{ab} \ge 2-a-b$

$\Rightarrow \dfrac{1}{\sqrt{ab}}-2+\sqrt{ab} +a-2\sqrt{ab}+b \ge0$

$\Rightarrow (\dfrac{1}{\sqrt[4]{ab}}-\sqrt[4]{ab})^2+(\sqrt{a}-\sqrt{b})^2 \ge0$ (luôn đúng)

Dấu bằng xảy ra khi $a=b=c=d=1$


Bài viết đã được chỉnh sửa nội dung bởi KietLW9: 22-03-2022 - 22:20


#53
KietLW9

KietLW9

    Đại úy

  • Điều hành viên THCS
  • 1737 Bài viết

Bài 42: Gặp rồi

Giả sử $a+b\le c+d$

$\dfrac{1}{ab}+\dfrac{1}{cd}\ge\dfrac{a^2+b^2+c^2+d^2}{2}$

$\Rightarrow \dfrac{1}{ab}-2+ab+\dfrac{1}{cd}-2+cd\ge\dfrac{a^2+b^2+c^2+c^2+2ab+2cd}{2}-4$

$\Rightarrow (\dfrac{1}{\sqrt{ab}}-\sqrt{ab})^2+(\dfrac{1}{\sqrt{cd}}-\sqrt{cd})^2\ge \dfrac{(a+b)^2+(c+d)^2}{2}-\dfrac{(a+b+c+d)^2}{4}$

$\Rightarrow (\dfrac{1}{\sqrt{ab}}-\sqrt{ab})^2 \ge \dfrac{(a+b-c-d)^2}{4}$

Vì $a+b\le c+d \Rightarrow a+b+c+d\le0;2\ge a+b \ge 2\sqrt{ab} \Rightarrow \sqrt{ab}\le 1 \Rightarrow \dfrac{1}{\sqrt{ab}}-\sqrt{ab} \ge0 $

Vậy ta cần chứng minh:

$\dfrac{1}{\sqrt{ab}}-\sqrt{ab} \ge \dfrac{c+d-a-b}{2}$

$\Rightarrow \dfrac{1}{\sqrt{ab}}-\sqrt{ab} \ge 2-a-b$

$\Rightarrow \dfrac{1}{\sqrt{ab}}-2+\sqrt{ab} +a-2\sqrt{ab}+b \ge0$

$\Rightarrow (\dfrac{1}{\sqrt[4]{ab}}-\sqrt[4]{ab})^2+(\sqrt{a}-\sqrt{b})^2 \ge0$ (luôn đúng)

Dấu bằng xảy ra khi $a=b=c=d=1$

Lời giải.

Ta cần chứng minh: $ab+\frac{1}{ab}+cd+\frac{1}{cd}\geqslant \frac{(a+b)^2+(c+d)^2}{2}$ Đặt $x = a+b,y=c+d$ thì $x+y=4$ và $x,y>0$. Vì trong 2 số $x,y$ luôn có một số lớn hơn hoặc bằng hai, luôn có một số bé hơn hoặc bằng 2 nên ta có thể giả sử $x\geqslant 2,y\leqslant 2$

Ta có: $ab+\frac{1}{ab}\geqslant 2$

Và $\frac{(c+d)^2-4cd}{cd(c+d)^2}\geqslant \frac{(c+d)^2-4cd}{4}\Rightarrow cd+\frac{1}{cd}\geqslant \frac{4}{y^2}+\frac{y^2}{4}$

Ta cần chứng minh: $2+\frac{4}{y^2}+\frac{y^2}{4}\geqslant \frac{(4-y)^2+y^2}{2}\Leftrightarrow (2-y)^3(3y+2)\geqslant 0$

Bất đẳng thức cuối đúng nên ta có điều phải chứng minh

Đẳng thức xảy ra khi $a=b=c=d=1$


Trong cuộc sống không có gì là đẳng thức , tất cả đều là bất đẳng thức  :ukliam2:   :ukliam2: 

 

 

$\text{LOVE}(\text{KT}) S_a (b - c)^2 + S_b (c - a)^2 + S_c (a - b)^2 \geqslant 0\forall S_a,S_b,S_c\geqslant 0$

 

 

 


#54
KietLW9

KietLW9

    Đại úy

  • Điều hành viên THCS
  • 1737 Bài viết

Bài 31: 

Đặt $a=\dfrac{1}{x};b=\dfrac{1}{y};c=\dfrac{1}{z} \Rightarrow xyz=1$

$\Rightarrow \sqrt{\dfrac{ab}{bc^2+1}}=\sqrt{\dfrac{ab}{bc(c+a)}}=\sqrt{\dfrac{a}{c(c+a)}}=\dfrac{x}{\sqrt{x+y}}$

Tương tự, có:

$VT=\dfrac{x}{\sqrt{x+y}}+\dfrac{y}{\sqrt{y+z}}+\dfrac{z}{\sqrt{z+x}}$

$\le \sqrt{(x+y+z)(\dfrac{x}{x+y}+\dfrac{y}{y+z}+\dfrac{z}{z+x})}$

$=\sqrt{\dfrac{(x+y+z) x}{x+y}+\dfrac{(x+y+z) y}{y+z}+\dfrac{(x+y+z) z}{z+x}}$

$=\sqrt{x+\dfrac{z x}{x+y}+y+\dfrac{x y}{y+z}+z+\dfrac{y z}{z+x}}$

$=\sqrt{x+\dfrac{z x y}{(x+y) y}+y+\dfrac{x y z}{(y+z) z}+z+\dfrac{y z x}{(z+x) x}}$

$\le \sqrt{x+\dfrac{z  \frac{1}{4}(x+y)^{2}}{(x+y) y}+y+\dfrac{x  \frac{1}{4}(y+z)^{2}}{(y+z) z}+z+\dfrac{y  \frac{1}{4}(z+x)^{2}}{(z+x) x}}$

$=\sqrt{x+\dfrac{z(x+y)}{4 y}+y+\dfrac{x(y+z)}{4 z}+z+\dfrac{y(z+x)}{4 x}}$

$=\sqrt{\dfrac{1}{4}\left(\dfrac{x y}{z}+\dfrac{y z}{x}+\dfrac{z x}{y}\right)+\frac{5}{4}(x+y+z)}$

$=\sqrt{\dfrac{1}{4}\left[(x y)^{2}+(y z)^{2}+(z x)^{2}\right]+\dfrac{5}{4}(x+y+z) x y z}$

 

Đến đây ta cần chứng minh $VP=\dfrac{a+b+c}{\sqrt{2}}=\dfrac{xy+yz+zx}{\sqrt{2}}\ge\sqrt{\dfrac{1}{4}\left[(x y)^{2}+(y z)^{2}+(z x)^{2}\right]+\dfrac{5}{4}(x+y+z) x y z}$

$\Rightarrow \dfrac{(xy+yz+zx)^2}{2} \ge \dfrac{1}{4}\left[(x y)^{2}+(y z)^{2}+(z x)^{2}\right]+\dfrac{5}{4}(x+y+z) x y z$

$\Rightarrow (xy)^2+(yz)^2+(zx)^2 \ge xy.yz+yz.zx+zx.xy$ (luôn đúng)

Dấu bằng xảy ra khi $a=b=c=1$

Câu hỏi được đặt ra. Vì sao $\sqrt{\dfrac{a}{c(c+a)}}=\dfrac{x}{\sqrt{x+y}}$ ?


Trong cuộc sống không có gì là đẳng thức , tất cả đều là bất đẳng thức  :ukliam2:   :ukliam2: 

 

 

$\text{LOVE}(\text{KT}) S_a (b - c)^2 + S_b (c - a)^2 + S_c (a - b)^2 \geqslant 0\forall S_a,S_b,S_c\geqslant 0$

 

 

 


#55
Do Linh An

Do Linh An

    Binh nhì

  • Thành viên mới
  • 14 Bài viết

Câu hỏi được đặt ra. Vì sao $\sqrt{\dfrac{a}{c(c+a)}}=\dfrac{x}{\sqrt{x+y}}$ ?

Chính xác thì $\sqrt{\dfrac{ab}{bc^2+1}}=\dfrac{z}{\sqrt{z+x}};\sqrt{\dfrac{bc}{ca^2+1}}=\dfrac{x}{\sqrt{x+y}};\sqrt{\dfrac{ca}{ab^2+1}}=\dfrac{y}{\sqrt{y+z}} $

Mình viết nhầm, nhưng không ảnh hưởng tới kết quả bài làm lắm


Bài viết đã được chỉnh sửa nội dung bởi KietLW9: 22-03-2022 - 22:26


#56
KietLW9

KietLW9

    Đại úy

  • Điều hành viên THCS
  • 1737 Bài viết

Bài 46: Cho $a,b,c$ là các số dương thỏa mãn $ab+bc+ca=3$. Chứng minh rằng: $\frac{1}{2a^3+1}+\frac{1}{2b^3+1}+\frac{1}{2c^3+1}\geqslant 1$

~~~~~~~~~~~~~~~~~~

Lời giải. (Hoang72)

Bổ đề: Nếu $x,y$ là các số thực dương thỏa mãn $xy\geqslant 1$ thì $\frac{1}{x^2+1}+\frac{1}{y^2+1}\geqslant \frac{2}{xy+1}$

Quay lại bài toán: Ta giả sử $ab\geqslant 1$ thì theo bổ đề: $\frac{1}{2a^3+1}+\frac{1}{2b^3+1}\geqslant \frac{2}{2\sqrt{a^3b^3}+1}\geqslant \frac{2}{\frac{2}{\sqrt{c^3}}+1}$ (Do từ giả thiết có $abc\leqslant 1$)

Vậy cần chứng minh: $\frac{2}{\frac{2}{\sqrt{c^3}}+1}+\frac{1}{2c^3+1}\geqslant 1$

Đặt $\sqrt{c^3}=t>0$ thì $\frac{2t}{t+2}+\frac{1}{2t^2+1}\geqslant 1\Leftrightarrow t(t-1)^2\geqslant 0$

Vậy bất đẳng thức được chứng minh

Đẳng thức xảy ra khi $a=b=c=1$

~~~~~~~~~~~~~~~~~~~~~

Bài tập tương tự: Cho $a,b,c$ là các số dương thỏa mãn $ab+bc+ca=3$. Chứng minh rằng: $\frac{1}{a^2+1}+\frac{1}{b^2+1}+\frac{1}{c^2+1}\geqslant  \frac{3}{2}$


Trong cuộc sống không có gì là đẳng thức , tất cả đều là bất đẳng thức  :ukliam2:   :ukliam2: 

 

 

$\text{LOVE}(\text{KT}) S_a (b - c)^2 + S_b (c - a)^2 + S_c (a - b)^2 \geqslant 0\forall S_a,S_b,S_c\geqslant 0$

 

 

 


#57
ha1

ha1

    Lính mới

  • Banned
  • 1 Bài viết
Nếu bạn muốn tìm hiểu thêm về cách giải toán cũng như những loại sữa tăng cân cho trẻ thì đừng quên liên hệ ngay cho chúng tôi nhé. 


#58
KietLW9

KietLW9

    Đại úy

  • Điều hành viên THCS
  • 1737 Bài viết

Bài 48: Cho $a,b,c$ là các số thực thỏa mãn $a+b+c=0$. Chứng minh rằng: $a^2b^2+b^2c^2+c^2a^2-6abc\geqslant -3$

~~~~~~~~~~~~~~~~

Lời giải.

Áp dụng bất đẳng thức $x^2+y^2\geqslant 2xy$, ta được: $a^2b^2+b^2c^2+c^2a^2=(ab+1)^2+(bc+1)^2+(ca+1)^2-2(ab+bc+ca)-3+(a+b+c)^2=\left [ (ab+1)^2+c^2 \right ]+\left [ (bc+1)^2+a^2 \right ]+\left [ (ca+1)^2+b^2 \right ]-3\geqslant 2c(ab+1)+2b(ca+1)+2b(ca+1)-3\Rightarrow a^2b^2+b^2c^2+c^2a^2-6abc\geqslant -3$

Đẳng thức xảy ra khi $a,b,c$ là hoán vị của bộ số $(2,-1,-1)$


Trong cuộc sống không có gì là đẳng thức , tất cả đều là bất đẳng thức  :ukliam2:   :ukliam2: 

 

 

$\text{LOVE}(\text{KT}) S_a (b - c)^2 + S_b (c - a)^2 + S_c (a - b)^2 \geqslant 0\forall S_a,S_b,S_c\geqslant 0$

 

 

 


#59
KietLW9

KietLW9

    Đại úy

  • Điều hành viên THCS
  • 1737 Bài viết

Bài 49: Cho $a,b,c$ là các số thực dương thỏa mãn $a^2+b^2+c^2=3$. Chứng minh rằng: $\sqrt{\frac{a^3}{b}}+\sqrt{\frac{b^3}{c}}+\sqrt{\frac{c^3}{a}}\geqslant 3$

~~~~~~~~~~~~~~~~~~

Lời giải.

Áp dụng bất đẳng thức $AM-GM$, ta được: $\sqrt{\frac{a^3}{b}}+\sqrt{\frac{b^3}{c}}+\sqrt{\frac{c^3}{a}}=\frac{a^2}{\sqrt{ab}}+\frac{b^2}{\sqrt{bc}}+\frac{c^2}{\sqrt{ca}}\geqslant \frac{2a^2}{a+b}+\frac{2b^2}{b+c}+\frac{2c^2}{c+a}=\left [ \frac{a^2+b^2}{a+b}+(a-b) \right ]+\left [ \frac{b^2+c^2}{b+c}+(b-c) \right ]+\left [ \frac{c^2+a^2}{c+a}+(c-a) \right ]=\frac{a^2+b^2}{a+b}+\frac{b^2+c^2}{b+c}+\frac{c^2+a^2}{c+a}$

Có một đánh giá rất đẹp sau: $(\sqrt{a^2+b^2}+\sqrt{b^2+c^2}+\sqrt{c^2+a^2})^2=2(a^2+b^2+c^2)+2\sqrt{(b^2+a^2)(b^2+c^2)}+2\sqrt{(c^2+a^2)(c^2+b^2)}+2\sqrt{(a^2+b^2)(a^2+c^2)}\geqslant 2(a^2+b^2+c^2)+2(b^2+ac)+2(c^2+ab)+2(a^2+bc)=3(a^2+b^2+c^2)+(a+b+c)^2=9+(a+b+c)^2\geqslant 6(a+b+c)$

Như vậy: $\sqrt{\frac{a^3}{b}}+\sqrt{\frac{b^3}{c}}+\sqrt{\frac{c^3}{a}}\geqslant \frac{a^2+b^2}{a+b}+\frac{b^2+c^2}{b+c}+\frac{c^2+a^2}{c+a}\geqslant \frac{(\sqrt{a^2+b^2}+\sqrt{b^2+c^2}+\sqrt{c^2+a^2})^2}{2(a+b+c)}\geqslant 3$

Đẳng thức xảy ra khi $a=b=c=1$


Trong cuộc sống không có gì là đẳng thức , tất cả đều là bất đẳng thức  :ukliam2:   :ukliam2: 

 

 

$\text{LOVE}(\text{KT}) S_a (b - c)^2 + S_b (c - a)^2 + S_c (a - b)^2 \geqslant 0\forall S_a,S_b,S_c\geqslant 0$

 

 

 


#60
KietLW9

KietLW9

    Đại úy

  • Điều hành viên THCS
  • 1737 Bài viết

Bài 50: Cho $1\leqslant a,b,c\leqslant 2$. Chứng minh rằng: $\frac{a}{a^2+ab+b^2}+\frac{b}{b^2+bc+c^2}+\frac{c}{c^2+ca+a^2}\leqslant \frac{1}{a+2b}+\frac{1}{b+2c}+\frac{1}{c+2a}$

Bài này khi nhìn vào thì sẽ thấy lạ ở chỗ cái giả thiết. Mà cái bất đẳng thức cần chứng minh hình như nó không sử dụng các số $1,2$ trong giả thiết lắm, đánh giá thường sẽ ngược dấu. Nhưng quan sát kĩ thì có thể thấy bất đẳng thức là thuần nhất, nên ta nghĩ ngay đến việc thiết lập một bất đẳng thức phụ, tạm gọi là UCT. BĐT phụ có dạng: $\frac{a}{a^2+ab+b^2}-\frac{1}{a+2b}\leqslant k(\frac{1}{a}-\frac{1}{b})$. Vì sao nó lại có dạng đó, đơn giản là làm cho xuất hiện nhân tử $a-b$ nhưng phải đồng bậc với bất đẳng thức đề ra, nên ta không chọn $a^3-b^3$ hay $a^4-b^4$. 

Câu hỏi đặt ra là liệu có thể làm như $UCT$ thông thường không? Câu trả lời là không. Vì nếu cho ngay $a=b$ thì hai vế bằng $0$ nên chả có ý nghĩa gì cả. Cho nên ta chỉ cho một biến có giá trị. Ta giả sử $a=1$ thì $\frac{1}{1+b+b^2}-\frac{1}{1+2b}\leqslant k(1-\frac{1}{b})\Leftrightarrow \frac{k(b-1)}{b}\geqslant \frac{b(1-b)}{(1+2b)(1+b+b^2)}$. Vì đồng bậc nên trên cơ sở lí thuyết không hoàn toàn thì: $k=\frac{-b^2}{(1+2b)(1+b+b^2)}=\frac{-1}{9}$ (Tức ta cho $b=1$, bằng với $a$)

Lúc này điều ta cần đã có, tức là BĐT phụ: $\boxed{\frac{a}{a^2+ab+b^2}-\frac{1}{a+2b}\leqslant \frac{1}{9}(\frac{1}{b}-\frac{1}{a})}$

Thật vậy, bất đẳng thức này tương đương: $\frac{(a-b)^2(a^2+4ab-2b^2)}{(a+2b)(a^2+ab+b^2)}\geqslant 0$ (Đúng bởi vì $a^2+4ab-2b^2=a^2+2b(2a-b)>0$ do $2a\geqslant 2\geqslant b$)

~~~~~~~~~~~~~~~~~~

Bài tập tương tự: Cho $1<a,b,c<3$. Chứng minh rằng: $\frac{2}{a+3b}+\frac{2}{b+3c}+\frac{2}{c+3a}\geqslant \frac{1}{a+b}+\frac{1}{b+c}+\frac{1}{c+a}$


Bài viết đã được chỉnh sửa nội dung bởi KietLW9: 26-03-2022 - 18:18

Trong cuộc sống không có gì là đẳng thức , tất cả đều là bất đẳng thức  :ukliam2:   :ukliam2: 

 

 

$\text{LOVE}(\text{KT}) S_a (b - c)^2 + S_b (c - a)^2 + S_c (a - b)^2 \geqslant 0\forall S_a,S_b,S_c\geqslant 0$

 

 

 





0 người đang xem chủ đề

0 thành viên, 0 khách, 0 thành viên ẩn danh